if z and (z+50) are supplement of each other find the value of z​

Answers

Answer 1

Answer:

z=65

Step-by-step explanation:

supplementary angles means sum of those angles is 180 degrees

so,

z+z+50=180

2z=130

z=65

I did the best I could, I'm 12 don't judge me.


Related Questions

Find an exact value of sin(17pi/12)

Answers

●✴︎✴︎✴︎✴︎✴︎✴︎✴︎✴︎❀✴︎✴︎✴︎✴︎✴︎✴︎✴︎✴︎✴︎●

   Hi my lil bunny!

❧⎯⎯⎯⎯⎯⎯⎯⎯⎯⎯⎯⎯⎯⎯⎯⎯⎯⎯⎯⎯⎯⎯⎯⎯⎯⎯⎯⎯⎯⎯⎯⎯⎯⎯⎯⎯⎯⎯☙

[tex]\frac{(17)(3.141593)}{12}[/tex]

= [tex]\frac{53.407075}{12}[/tex]

= [tex]4.45059[/tex]

❧⎯⎯⎯⎯⎯⎯⎯⎯⎯⎯⎯⎯⎯⎯⎯⎯⎯⎯⎯⎯⎯⎯⎯⎯⎯⎯⎯⎯⎯⎯⎯⎯⎯⎯⎯⎯⎯⎯☙

●✴︎✴︎✴︎✴︎✴︎✴︎✴︎✴︎❀✴︎✴︎✴︎✴︎✴︎✴︎✴︎✴︎✴︎●

If this helped you, could you maybe give brainliest..?

Also Have a great day/night!

❀*May*❀

I have an answer and explanation but I can't type so search up the question you asked and you should get an answer and explanation from s0cratic.


What is the midline equation of the function h(x) = -4 cos(5x - 9) - 7?

Answers

Answer: Midline equation: y = -7

Step-by-step explanation: This function is a sinusoidal function of the form:

y = a.cos(b(x+c))+d

Midline is a horizontal line where the function oscillates above and below.

In the sinusoidal function d represents its vertical shift. Midline is not influenced by any other value except vertical shift. For that reason,

Midline, for the function: [tex]h(x) = -4cos(5x-9) - 7[/tex] is y=d, i.e., [tex]y=-7[/tex]

Answer:

y=-7

Step-by-step explanation:

(G1) The distance from Flagstaff Arizona to
Tucson Arizona is 260 miles. Express this
distance in meters.
A. 418,418 meters
B. 419,000 meters
C. 126,200 meters
D. 260,000 meters

Answers

Answer:

A. 418, 418

Step-by-step explanation:

The formula to convert miles to meters is the following:

1 = 1,609.34

so for every 1 mile, you have 1,609.34 meters

so you take your distance in miles and multiply it by 1,609.34

d= 260 x 1,609.34

d = 418, 428.4

Which is a factor of: 2x2+10x+8 ?

Answers

Answer:

2 ( x+4) ( x+1)

Step-by-step explanation:

2x^2+10x+8

Factor out 2

2 ( x^2 +5x+4)

What two numbers  multiply to 4 and add to 5

4*1 = 4

4+1 = 5

2 ( x+4) ( x+1)

[tex] \large{ \underline{ \underline{ \bf{ \pink{To \: factorise}}}}}[/tex]

2x² + 10x + 8

Factorisation:

By middle term factorisation,

⇛ 2x² + 2x + 8x + 8

⇛ 2x(x + 1) + 8(x + 1)

⇛ (2x + 8)(x + 1)

⇛ 2(x + 4)(x + 1)

☃️ Now you can break it down and check which are the factors of the polynomial according to options.

━━━━━━━━━━━━━━━━━━━━

find the area of this figure to the nearest hundredth. Use 3.14 to approximate pi.

Answers

Answer:

86.28 ft²

Step-by-step explanation:

The figure given consists of a rectangle and a semicircle.

The area of the figure = area of rectangle + area of semicircle

Area of rectangle = [tex] l*w [/tex]

Where,

l = 10 ft

w = 8 ft

[tex] area = l*w = 10*8 = 80 ft^2 [/tex]

Area of semicircle:

Area of semicircle = ½ of area of a circle = ½(πr²)

Where,

π = 3.14

r = ½ of 8 = 4 ft

Area of semi-circle = ½(3.14*4) = 6.28 ft²

Area of the figure = area of rectangle + area of semi-circle = 80 + 6.28 = 86.28 ft² (nearest hundredth)

Answer:

the area of the figrue is 105.12

Step-by-step explanation:

area of rectangle A= l · w10 x 8= 80area of simi-circle= 1/2(3.14 x r²)1/2 x 3.14 x 4²=25.1280+25.12=105.12 (nearest Hundredth)

if G is the midpoint of FH, FG = 14x + 25 and GH = 73 - 2x, find FH.

Answers

Answer:

FH = 134

Step-by-step explanation:

From the question given:

G is the midpoint of FH

FG = 14x + 25

GH = 73 - 2x

FH =?

Next, we shall determine the value of x. The value of x can be obtained as follow:

Since G is the midpoint of FH, this implies that FG and GH are equal i.e

FG = GH

With the above formula, we can obtain the value of x as follow:

FG = 14x + 25

GH = 73 - 2x

x =?

FG = GH

14x + 25 = 73 - 2x

Collect like terms

14x + 2x = 73 - 25

16x = 48

Divide both side by 16

x = 48/16

x = 3

Next, we shall determine the value of FG and GH. These can be obtained as shown below:

FG = 14x + 25

x = 3

FG = 14x + 25

FG = 14(3) + 25

FG = 42 + 25

FG = 67

GH = 73 - 2x

x = 3

GH = 73 - 2x

GH = 73 - 2(3)

GH = 73 - 6

GH = 67

Finally, we shall determine FH as follow:

FH = FG + GH

FG = 67

GH = 67

FH = FG + GH

FH = 67 + 67

FH = 134

Therefore, FH is 134

Determine if the process appears to be within statistical control. If not, state the reason why not.
a. It does not appear to be within statistical control because there is an upward shift.
b. It appears to be within statistical control.
c. It does not appear to be within statistical control because there is an upward trend.
d. It does not appear to be within statistical control because there is increasing variation.

Answers

Answer:

c. It does not appear to be within statistical control because there is an upward trend.

Step-by-step explanation:

Statistical process control is a method for quality control which employs statistical method to monitor and control process. It ensures operation efficiency and ensuring required specification to reduce wastes in production lines. Here the process variation is out of control because the statistical control has an upward trend.

Which of the following is true about congruent figures?
They're the same shape and the same size.
They're the same size, but not the same shape.
They're not the same shape or size.
They're the same shape, but not the same size.​

Answers

Answer:

A

Step-by-step explanation:

congruent means they have the same shape and size. hope this helps :)

solve the following inequalities 7 x minus 5 / 8 x + 3 >4

Answers

Answer:

[tex]x> \frac{8}{51} [/tex]

Step-by-step explanation:

[tex]7x - \frac{5}{8} x + 3>4[/tex]

Bring constants to one side, simplify:

[tex] \frac{51}{8} x>4 - 3 \\ \frac{51}{8} x>1 \\ x>1 \div \frac{51}{8} \\ x>1 \times \frac{8}{51} \\ x> \frac{8}{51} [/tex]

*Note that the inequality sign only changes when you divide the whole inequality by a negative number.

Answer:

[tex]x>\frac{8}{51}[/tex]

Step-by-step explanation:

[tex]7x-\frac{5}{8}x+3>4\\\mathrm{Subtract\:}3\mathrm{\:from\:both\:sides}\\7x-\frac{5}{8}x+3-3>4-3\\\mathrm{Simplify}\\7x-\frac{5}{8}x>1\\\mathrm{Multiply\:both\:sides\:by\:}8\\7x\times \:8-\frac{5}{8}x\times \:8>1\times \:8\\\mathrm{Simplify}\\56x-5x>8\\51x>8\\\mathrm{Divide\:both\:sides\:by\:}51\\\frac{51x}{51}>\frac{8}{51}\\\\x>\frac{8}{51}[/tex]

I hope it helps :)

13,226 divided by 29

Answers

13226/29= 456.068965517

One of two small restaurants is chosen at random with equally likely probability, and then an employee is chosen at random from the chosen restaurant. Restaurant #1 has 10 full-timers and 6 part-timers. Restaurant #2 has 7 full-timers and 9 part-timers. What is the probability that Restaurant #1 was chosen at random, given that a full-time employee was chosen? Your answers should be rounded to 4 digits after the decimal.

Answers

Answer:

P(1 |F) = 10/17

Step-by-step explanation:

Let events

1 = restaurant 1

2 = restaurant 2

F = full-time worker chosen

P = part-time worken chosen

P(1 and F) = 1/2 * 10/16 = 5/16

P(2 and F) = 1/2 * 7/16 = 7/32

P( (1 or 2) and F ) = P(F) = 5/16+7/32 = 17/32

P(1 | F)          Probability of choosing restaurant 1 given a full-time was chosen

= P(1 and F) / P(F)

= 5/16  / (17/32)

= 5/16 * 32/17

= 10 / 17

normal population has a mean of 63 and a standard deviation of 13. You select a random sample of 25. Compute the probability that the sample mean is: (Round your z values to 2 decimal places and final answers to 4 decimal places): Greater than 65.

Answers

Answer:

0.2207

Step-by-step explanation:

Here, we want to find the probability that the sample mean is greater than 25.

What we use here is the z-scores statistic

Mathematically;

z-score = (x-mean)/SD/√n

From the question;

x = 65, mean = 63, SD = 13 and n = 25

Plugging these values in the z-score equation, we have

Z-score = (65-63)/13/√25 = 2/13/5 = 0.77

So the probability we want to calculate is ;

P(z > 0.77)

This can be obtained from the standard normal distribution table

Thus;

P(z > 0.77) = 0.22065 which is 0.2207 to 4 d.p

Translate and solve: 54 greater than x is greater than 216

Answers

Answer:

x >162

Step-by-step explanation:

x+54 > 216

Subtract 54 from each side

x+54-54 > 216 - 54

x >162

Answer:

[tex]\huge \boxed{{x>162}}[/tex]

Step-by-step explanation:

[tex]x+54 > 216[/tex]

[tex]\sf Subtract \ 54 \ from \ both \ parts.[/tex]

[tex]x+54 -54> 216-54[/tex]

[tex]x>162[/tex]

What is the constant of variation, k, of the line y=kx through (3,18) and (5,30)? 3 6

Answers

Answer:

6

Step-by-step explanation:

The constant of variation is the slope

k = (y2-y1)/(x2-x1)

  = (30-18)/(5-3)

   =12/2

   = 6

The value of constant of variation, k, is,

⇒ k = 6

What is Equation of line?

The equation of line in point-slope form passing through the points

(x₁ , y₁) and (x₂, y₂) with slope m is defined as;

⇒ y - y₁ = m (x - x₁)

Where, m = (y₂ - y₁) / (x₂ - x₁)

Here, the constant of variation, k, of the line y = kx through (3,18) and (5,30)

Since, The constant of variation is the slope,

Hence, We get;

k = (y₂ - y₁)/(x₂ - x₁)

 = (30 - 18)/(5 - 3)

  = 12/2

  = 6

Thus, the value of constant of variation, k, is,

⇒ k = 6

Learn more about the equation of line visit:

https://brainly.com/question/18831322

#SPJ7

(a) A survey of the adults in a town shows that 8% have liver problems. Of these, it is also found that 25% are heavy drinkers, 35% are social drinkers and 40% are non-drinkers. Of those that did not suffer from liver problems, 5% are heavy drinkers, 65% are social drinkers and 30% do not drink at all. An adult is chosen at random, what is the probability that this person i. Has a liver problems?

Answers

Answer:

The probability that the selected adult has liver problems is 0.08

Step-by-step explanation:

In this question, from the data given, we want to calculate the probability that an adult selected at random has liver problems.

Let E(L) be the event that an adult has liver problems.

The probability is directly obtainable from the question and it is given as 8%

Thus, the probability that the selected adult has liver problems; P(L) = 8% = 8/100 = 0.08

What is the exact distance from (−1, 4) to (6, −2)? square root of 80. units square root of 82. units square root of 85. units square root of 89. units

Answers

Answer:

[tex]\sqrt{85}[/tex].

Step-by-step explanation:

[tex]x[/tex]-coordinates:

First point: [tex]-1[/tex].Second point: [tex]6[/tex].Difference: [tex]|-1 - 6| = |-7| = 7[/tex].

[tex]y[/tex]-coordinates:

First point: [tex]4[/tex].Second point: [tex]-2[/tex].Difference: [tex]|4 - (-2)| = |6| = 6[/tex].

Refer to the diagram attached. Consider these two points as the two end points of the hypotenuse of a right triangle. The lengths of the two legs are equal to:

the difference between the two [tex]x[/tex]-coordinates, [tex]7[/tex], and the difference between the two [tex]y[/tex]-coordinates, [tex]6[/tex].

Apply Pythagorean Theorem to find the length of the hypotenuse (which is equal to the distance between the two points in question.)

[tex]\begin{aligned}\text{Hypotenuse} &= \sqrt{(\text{First Leg})^2 + (\text{Second Leg})^2} \\ &= \sqrt{7^2 + 6^2} \\ &= \sqrt{85}\end{aligned}[/tex].

Answer:

C

Step-by-step explanation:

Yelena needs to swim a total of 8 miles this
week. So far, she swam 5 miles. Use the
equation 5 + m=8 to find how many more
miles Yelena needs to swim.

Answers

Answer:

3 miles

Step-by-step explanation:

5 + m=8

Subtract 5 from each side

5-5 + m=8-5

m = 3

She needs to swim 3 more miles

Answer:

Yelena needs to swim 3 more miles

Step-by-step explanation:

You need to solve for the variable "m", which represents the miles. Based on the information, Yelena swam 5 miles and she needs to swim 8. Solve:

[tex]5+m=8[/tex]

To find the value of m, you need to isolate it on one side of the equation. To do this, you need to get the 8 and 5 on the same side of the equal operation. For this, you need to use reverse operations. This undoes the value from one side and does the same on the other, keeping the equation balanced. Since we have a "positive 5", we take the opposite, which would be a "negative 5". So subtract 5 from both sides of the equation:

[tex]5-5+m=8-5[/tex]

Simplify. The 5's cancel each other out, leaving 0. 8-5 is 3:

[tex]m=3[/tex]

The total miles left that Yelena needs to swim is 3 miles.

:Done

Given the following hypotheses: H0: μ = 490 H1: μ ≠ 490 A random sample of 15 observations is selected from a normal population. The sample mean was 495 and the sample standard deviation 9. Using the 0.01 significance level:
a.) State the decision rule.
b.) Compute the value of the test statistic.
c.) What is your decision regarding the null hypothesis?

Answers

Answer:

We conclude that the population mean is equal to 490.

Step-by-step explanation:

We are given that a random sample of 15 observations is selected from a normal population. The sample mean was 495 and the sample standard deviation 9.

Let [tex]\mu[/tex] = population mean.

So, Null Hypothesis, [tex]H_0[/tex] : [tex]\mu[/tex] = 490      {means that the population mean is equal to 490}

Alternate Hypothesis, [tex]H_A[/tex] : [tex]\mu\neq[/tex] 490     {means that the population mean is different from 490}

The test statistics that will be used here is One-sample t-test statistics because we don't know about population standard deviation;

                               T.S.  =  [tex]\frac{\bar X-\mu}{\frac{s}{\sqrt{n} } }[/tex]  ~ [tex]t_1_4[/tex]

where, [tex]\bar X[/tex] = sample mean = 495

            s = sample standard deviation = 9

             n = sample of observations = 15

So, the test statistics =   [tex]\frac{495-490}{\frac{9}{\sqrt{15} } }[/tex]  ~ [tex]t_1_4[/tex]

                                     =  2.152

The value of t-test statistics is 2.152.

Now, at a 0.01 level of significance, the t table gives a critical value of -2.977 and 2.977 at 14 degrees of freedom for the two-tailed test.

Since the value of our test statistics lies within the range of critical values of t, so we have insufficient evidence to reject our null hypothesis as the test statistics will not fall in the rejection region.

Therefore, we conclude that the population mean is equal to 490.

Geometry pls help !!! Find the value of AB.
AB = [?]

Answers

Answer:

AB = 16 Units

Step-by-step explanation:

In the given figure, CD is the diameter and AB is the chord of the circle.

Since, diameter of the circle bisects the chord at right angle.

Therefore, AE = 1/2 AB

Or AB = 2AE...(1)

Let the center of the circle be given by O. Join OA.

OA = OD = 10 (Radii of same circle)

Triangle OAE is right triangle.

Now, by Pythagoras theorem:

[tex] OA^2 = AE^2 + OE^2 \\

10^2 = AE^2 + 6^2 \\

100= AE^2 + 36\\

100-36 = AE^2 \\

64= AE^2 \\

AE = \sqrt{64}\\

AE = 8 \\

\because AB = 2AE..[From \: equation\: (1)] \\

\therefore AB = 2\times 8\\

\huge \purple {\boxed {AB = 16 \: Units}} [/tex]

Time spent using e-mail per session is normally distributed with a mean = to 8 minutes and standard deviation = 2minutes. If a random samples of 36 sessions were selected, the computed sample standard deviation would be

a. 0.25
b. 0.3333
c. 0.42
d. 0.48

Answers

Answer:

The correct option is (b) 0.3333.

Step-by-step explanation:

The standard deviation of the sampling distribution of sample mean [tex](\bar x)[/tex] is known as the standard error [tex](\sigma_{\bar x})[/tex].

The standard error is given as follows:

[tex]\sigma_{\bar x}=\frac{\sigma}{\sqrt{n}}[/tex]

The information provided is:

[tex]\mu=8\\\\\sigma=2\\\\n=36[/tex]

Compute the standard deviation of the sample mean as follows:

[tex]\sigma_{\bar x}=\frac{\sigma}{\sqrt{n}}[/tex]

    [tex]=\frac{2}{\sqrt{36}}\\\\=\frac{2}{6}\\\\=\frac{1}{3}\\\\=0.3333[/tex]

Thus, the standard deviation of the sample mean is 0.3333.

a lottery offers one $1000 prize one $500 and two $50 prizes. one thousand tickets are sold at $2.50. what is the expectived profit

Answers

Answer:

 $900

Step-by-step explanation:

To begin with let us estimate the total cash value of the prices

$1000 x 1= 1000

$500 x 1=  500

$50 x 2= 100

Total = $1600

Now let us calculate the total cost of tickets sold at $2.50 per tickets for 1000 tickets

2.5*1000= $2,500

Assuming worse case that the lottery had winners in all three categories and i.e the total prices given out is $1600

Then the expected profit is = $2,500-$1600= $900



x = 4: 3x3 - 2x2 +10

Answers

Answer:

170

Step-by-step explanation:

3(4)³ - 2(4)² + 10

192 - 32 + 10 = 170

una compañía sabe que si produce "x" unidades mensuales su utilidad "u" se podría calcular con la expresión:


u(x)=-0.04x^2+44x-4000


donde "u" se expresa en dólares. Determine la razón del cambio promedio de la utilidad cuando el nivel de producción cambia de 600 a 620 unidades mensuales. Recuerde que la pendiente de la recta secante a la gráfica de la función representa a la razón de cambio promedio.


porfavor alguien que me explique el procedimiento :(

Answers

Answer:

Δf(u) /Δx  = 92,8    ( razón de cambio promedio)

Step-by-step explanation:

La expresión de la utilidad de la empresa u(x) en función de la cantidad de unidades producidas "x" es:

u(x) = 0,04*x² + 44*x  -4000

Entonces la razón de cambio promedio en un intervalo (a ; b)  en este caso ( 620 ;  600 ) viene dada por la expresión:

Δf(x)/ Δx  =  [ f(b)  -  f(a) ]/( b - a )

en donde  f(b)  y f(a) se obtienen por sustitución de los valores a y b es decir  600 y 620 respectivamente en la función f(x) = u(x) entonces

Δf(u) /Δx  =  [ u(b)  -  u(a) ]/( b -a )        (1)

u(b) =  0,04*(620)² + 44*(620) - 4000

u(b) = 15376 + 27280 -4000

u(b) = 2656 unidades

u(a)  = 0,04* (600)²  +  44* 600  - 4000

u(a)  = 14400  +  26400  - 4000

u(a) = 800

Sustituyendo esos valores en la ecuación 1

Δf(u) /Δx  =  2656 - 800 / 620 - 600

Δf(u) /Δx  = 92,8

PLEASE HELP ! (2/5) -50 POINTS -

Answers

Answer:

symmetric

Step-by-step explanation:

it kind of evenly falls to the left and right from the highest value in the middle

skewed would be different and would look like a straight line not a quadratic equation

C) symmetric distribution because the mean, median, mode happen at about the same point

coefficient of 8x+7y

Answers

Answer: I’m pretty sure it is 8 and 7 because those are the numbers multiplied with variables.

Answer:

8

Step-by-step explanation:

Identify the exponents on the variables in each term, and add them together to find the degree of each term.

8x→1

7y→1

The largest exponent is the degree of the polynomial.

1

The leading term in a polynomial is the term with the highest degree.

8x

The leading coefficient of a polynomial is the coefficient of the leading term.

____________________________________________________________

The leading term in a polynomial is the term with the highest degree.

8x

The leading coefficient in a polynomial is the coefficient of the leading term.

8

List the results.

Polynomial Degree: 1

Leading Term: 8x

Leading Coefficient: 8

Hope This Helps!!!

What is the sum of the geometric sequence?

Answers

Answer:

B. 259

Step-by-step explanation:

6^(i - 1) for i = 1 to 4

sum = 6^(1 - 1) + 6^(2 - 1) + 6^(3 - 1) + 6^(4 - 1) =

= 6^0 + 6^1 + 6^2 + 6^3

= 1 + 6 + 36 + 216

= 259

Answer: B. 259

A new fast-food firm predicts that the number of franchises for its products will grow at the rate dn dt = 6 t + 1 where t is the number of years, 0 ≤ t ≤ 15.

Answers

Answer:

The answer is "253"

Step-by-step explanation:

In the given- equation there is mistype error so, the correct equation and its solution can be defined as follows:

Given:

[tex]\bold{\frac{dn}{dt} = 6\sqrt{t+1}}\\[/tex]

[tex]\to dn= 6\sqrt{t+1} \ \ dt.....(a)\\\\[/tex]

integrate the above value:

[tex]\to \int dn= \int 6\sqrt{t+1} \ \ dt \\\\\to n= \frac{(6\sqrt{t+1} )^{\frac{3}{2}}}{\frac{3}{2}}+c\\\\\to n= \frac{(12\sqrt{t+1} )^{\frac{3}{2}}}{3}+c\\\\[/tex]

When the value of n=1 then t=0

[tex]\to 1= \frac{12(0+1)^{\frac{3}{2}}}{3}+c\\\\ \to 1= \frac{12(1)^{\frac{3}{2}}}{3}+c\\\\\to 1-\frac{12}{3}=c\\\\\to \frac{3-12}{3}=c\\\\\to \frac{-9}{3}=c\\\\\to c=-3\\[/tex]

so the value of  n is:

[tex]\to n= \frac{(12\sqrt{t+1} )^{\frac{3}{2}}}{3}-3\\\\[/tex]

when we put the value t= 15 then,

[tex]\to n= \frac{(12\sqrt{15+1} )^{\frac{3}{2}}}{3}-3\\\\\to n= \frac{(12\sqrt{16} )^{\frac{3}{2}}}{3}-3\\\\\to n= \frac{(12\times 64)}{3}-3\\\\\to n= (4\times 64)-3\\\\\to n= 256-3\\\\\to n= 253[/tex]

A parent increases a child’s monthly allowance by 20% each year. If the allowance is $8 per month now, in about how many years will it take to reach $20 per month? Use the equation 20 = 8(1.2)x to solve the problem. Round to the nearest year. 1 year 5 years 2 years 16 years

Answers

Answer:

6 years

Step-by-step explanation:

A parent increases a child’s monthly allowance by 20% each year. If the allowance is $8 per month now. This is an exponential function, An exponential function is given by:

[tex]y=ab^x[/tex]

Let x be the number of years and y be the allowance. The initial allowance is $8, this means at x = 0, y = 8

[tex]y=ab^x\\8=ab^0\\a=8[/tex]

Since it increases by 20% each year, i.e 100% + 20% = 1 + 0.2 = 1.2. This means that b = 1.2

Therefore:

[tex]y=ab^x\\y=8(1.2^x) \\[/tex]

To find the number of years will it take to reach $20 per month, we substitute y = 20 and find x

[tex]20=8(1.2)^x\\20/8=1.2^x\\1.2^x=2.5\\Taking \ natural\ log\ of \ both\ sides:\\ln(1.2^x)=ln2.5\\xln(1.2)=0.9163\\x=0.9163/ln(1.2)\\x=5.026[/tex]

x = 6 years to the nearest year

Answer:

5 years

Step-by-step explanation:444

1. What is the value of (1/2)^3?
O A. 76
O B. 119
O C.12
O D. 18​

Answers

Answer:

1/2 to the power of 3= 1/8

Step-by-step explanation:

1/2*1/2=1/4

1/4*1/2=1/8

d?

[tex]\huge\text{Hey there!}[/tex]


[tex]\mathsf{(\dfrac{1}{2})^3}[/tex]

[tex]\mathsf{= \dfrac{1}{2}^3}[/tex]

[tex]\mathsf{= \dfrac{1}{2} \times \dfrac{1}{2} \times \dfrac{1}{2}}[/tex]

[tex]\mathsf{= \dfrac{1 \times 1 \times 1}{2 \times 2 \times 2}}[/tex]

[tex]\mathsf{\mathsf{= \dfrac{1 \times 1} {4 \times 2}}}[/tex]

[tex]\mathsf{= \dfrac{1}{8}}[/tex]



[tex]\huge\text{Therefore your answer should be:}[/tex]

[tex]\huge\boxed{\mathsf{Option\ D.\ \dfrac{1}{8}}}\huge\checkmark[/tex]


[tex]\huge\text{Good luck on your assignment \& enjoy your day!}[/tex]



~[tex]\frak{Amphitrite1040:)}[/tex]

WILL GIVE BRAINLEST PLEASE!!!!!!!! Jenny has some tiles in a bag. The tiles are of three different colors: purple, pink, and orange. Jenny randomly pulls a tile out of the bag, records the color, and replaces the tile in the bag. She does this 50 times. The results are recorded in the given table: Color of Tile Purple Pink Orange Number of times the tile is drawn 6 18 26 What is the experimental probability that Jenny will pull out a purple tile? fraction 6 over 50 fraction 44 over 50 fraction 6 over 44 fraction 18 over 44

Answers

Answer:

6/50

Step-by-step explanation:

There are 50 tiles

6 purple

18 pink

26 orange

P( purple) = purple/ total

                = 6/50

Other Questions
A quote (from a song or a book) that shows who you are is... What is the value of x?7 7 square root 21414 square root 2 Discuss the Great Compromise and identify the feature of todays American government that the Great Compromise resulted in. [MO1.2] What is foreign aid ? Why do you like or dislike foreign aid? Present your critical analysis in eight points. What is the correct alternate hypothesis if the pilots' average gain score due to alcohol is indicated in the hypothesis statement by Describe the following as a (a) distance (b) speed (c) acceleration (d) force How many pepperoni pizzas did they buy if they bought 6 cheese pizzas Complete parts (a) through (c) below. (a) Determine the critical value(s) for a right-tailed test of a population mean at the alpha = 0.10 level of significance with 15 degrees of freedom. (b) Determine the critical value(s) for a left-tailed test of a population mean at the alpha = 0.01 level of significance based on a sample size of n = 20. (c) Determine the critical value(s) for a two-tailed test of a population mean at the alpha = 0.05 level of significance based on a sample size of n = 14. Find the slope and the y-intercept of the graph of the linear equation.y=-7x+2A) slope: - 1 y-intercept: 2B) slope: 2; y-intercept: -7C) slope: -7; y-intercept: 2D) slope: 2; y-intercept: -7 Lisa built a rectangular flower garden that is 4 meters wide and has a perimeter of 26 meters.What is the length of Lisa's flower garden? andrew is a software professional. he wants to join a group of software professionals on a networking site to remain updated on latest software trends. which network site should andrew join? If you were to climb to the top of the highest peak in the Himalayas and look northeast, which of these physical features would you looking at? O E. $68.00QUESTION 163.04 poiYou are a school photographer taking individual and class pictures for 2 classes of21 students each. On average, each individual picture takes 3 minutes and a classpicture takes 10 minutes. About how long should it take you to get all of thepictures?O A. 1 hour 3 minutesB. 1 hour 13 minutesOC. 2 hours 6 minutesOD. 2 hours 16 minutesO E. 2 hours 26 minutes what is meant by energy?list it various sources Billy has x marbles. Write an expression for the number of marbles the following have a) Charlie has 5 more than Billy b) Danny has 8 fewer than Billy c) Eric has three times as many as Billy Please find the perimeter! The highest mountain on mars is olympus mons, rising 22000 meters above the martian surface. If we were to throw an object horizontaly off the mountain top, how long would it take to reach the surface? (Ignore atmospheric drag forces and use gMars=3.72m/s^2a. 2.4 minutesb. 0.79 minutesc. 1.8 minutesd. 3.0 minutes The graph of the function f(x) = (x 3)(x + 1) is shown.On a coordinate plane, a parabola opens up. It goes through (negative 1, 0), has a vertex at (1, negative 4), and goes through (3, 0).Which describes all of the values for which the graph is positive and decreasing?all real values of x where x < 1all real values of x where x < 1all real values of x where 1 < x < 3all real values of x where x > 3 The direction of the functional group is called? after allowing 20 percent discount on the marked price of a radio 15 percent vat is levied on it , if its price become rs 22080 ,what amount was levied in the vat